Find the minimum sample size needed to estimate the percentage of Democrats who have a sibling. Use a 0.1 margin of error, use a confidence level of 98%, and use the results from a prior Harris poll that gave a confidence interval of (0.44, 0.51) for the proportion of Democrats who have a sibling.

Answers

Answer 1

Answer:

The minimum sample size is  [tex]n =135[/tex]

Step-by-step explanation:

From the question we are told that

   The confidence interval is [tex]( lower \ limit = \ 0.44,\ \ \ upper \ limit = \ 0.51)[/tex]

    The margin of error is  [tex]E = 0.1[/tex]

   

Generally the sample  proportion can be mathematically evaluated as

     [tex]\r p = \frac{ upper \ limit + lower \ limit }{2}[/tex]

    [tex]\r p = \frac{ 0.51 + 0.44}{2}[/tex]

    [tex]\r p = 0.475[/tex]

Given that the confidence level is  98% then the level of significance can be mathematically evaluated as

         [tex]\alpha = 100 - 98[/tex]

        [tex]\alpha = 2\%[/tex]

        [tex]\alpha =0.02[/tex]

Next we obtain the critical value of  [tex]\frac{\alpha }{2}[/tex] from the normal distribution table  

   The value is

        [tex]Z_{\frac{\alpha }{2} } = 2.33[/tex]

Generally the minimum sample size is evaluated as

      [tex]n =[ \frac { Z_{\frac{\alpha }{2} }}{E} ]^2 * \r p (1- \r p )[/tex]

     [tex]n =[ \frac { 2.33}{0.1} ]^2 * 0.475(1- 0.475 )[/tex]

     [tex]n =135[/tex]


Related Questions

A Ferris wheel has a diameter of 42 feet it rotates 3 times per minute approximately how far will a passenger travel during a 5 minute ride

Answers

Answer:

1978.2 or 630π feet

Step-by-step explanation:

The Ferris wheel will rotate 3 * 5 = 15 times during the 5 minute ride and the radius is 42 / 2 = 21 feet. Since C = 2πr, r = 21 and π ≈ 3.14, C = 2 * 3.14 * 21 = 131.88. However, this only accounts for one rotation so the answer is 131.88 * 15 = 1978.2 or 630π feet.

Pamela drove her car 99 kilometers and used 9 liters of fuel. She wants to know how many kilometers (k)left parenthesis, k, right parenthesis she can drive with 12 liters of fuel. She assumes the relationship between kilometers and fuel is proportional.


How many kilometers can Pamela drive with 12 liters of fuel?

Answers

Answer:

132 kilo meters

Step-by-step explanation:

Pro por tions:

9 lite rs ⇒ 99 km

12 lite rs  ⇒  P km

P = 99*12/9

P = 132 km

Answer:

132

Step-by-step explanation:

give person above brainliest :))

The force of gravity on an object varies directly with its mass. The constant of variation due to gravity is 32.2 feet per second squared. Which equation represents F, the force on an object due to gravity according to m, the object’s mass? F = 16.1m F = F equals StartFraction 16.1 Over m squared EndFraction. F = 32.2m F = F equals StartFraction 32.2 Over m squared EndFraction.

Answers

Answer:

F = 32.2m

Step-by-step explanation:

According to newton second law, the force of gravity on an object varies directly with its mass and it is expressed mathematically as Fαm i.e

F = mg where;

F is the force of gravity

m is the mass of the body

g is the proportionality constant known as the acceleration due to gravity.

If the constant of variation due to gravity is 32.2ft/s², the equation that represents F, the force on an object due to gravity according to m, the object’s mass can be gotten by substituting g = 32.2 into the formula above according to the law as shown;

F = m*32.2

F =32.2m

Hence the required equation is F = 32.2m

the ration of men to women in a certain factory is 3 to 4. there are 204 men. how many workers are there?

Answers

Answer:

476 workers

Step-by-step explanation:

Men: women : total

3          4           3+4 = 7

We want 204 men

204/3 =68

Multiply each by 68

Men: women : total

3*68    4*68      7*68

204        272      476

Answer:

There are 476 workers

Step-by-step explanation:

Can someone help??????????

Answers

The answer is b because are congruent

Answer:

(C) 1 and 3

Step-by-step explanation:

Corresponding angles are angles that are at the same corner at the different intersections.

We can see that 1 is on the bottom right corner of the bottom line, now we need to see what angle is at the bottom right corner of the top line?

That's 3.

So 1 and 3 are congruent because they are corresponding.

Hope this helped!

A package of 8-count AA batteries costs $6.40. A package of 20-count AA batteries costs $15.80. Which statement about the unit prices is true?

Answers

Answer:

The unit price of the 20 pack is $0.79 and the unit price for the 8 pack is $0.80.

Step-by-step explanation:

Simply Take the price of the pack of batteries divided by the number within the pack.

$6.40 / 8 == $0.80

$15.80 / 20 == $0.79

Cheers.

The question is incomplete. You can find the missing content below.

A package of 8-count AA batteries costs $6.40. A package of 20-count Of batteries costs $15.80. Which statement about the unit prices is true?

A) The 8-count pack of AA batteries has a lower unit price of $0.79 per battery.

B) The 20-count pack of AA batteries has a lower unit price of $0.80 per battery.

C) The 8-count pack of AA batteries has a lower unit prices of $0.80 per battery.

D) The 20-count pack of AA batteries has a lower unit price of $0.79 per battery.

The correct option is Option D: The 20-count pack of AA batteries has the lower price of $0.79 per battery.

What is inequality?

Inequality is the relation between two numbers or variables or expressions showing relationships like greater than, greater than equals to, lesser than equals to, lesser than, etc.

For example 2<9

A package of 8-count AA batteries has cost = $6.40.

cost per unit count AA batteries will be= total cost of AA batteries/ number of AA batteries

= $6.40/8= $0.8

A package of 20-count AA batteries has cost = $15.80.

cost per unit count AA batteries will be= total cost of AA batteries/ number of AA batteries

= $15.80/20= $0.79

As 0.79<0.8

cost of 20-count AA batteries <  cost of 8-count AA batteries

Therefore the correct option is Option D: The 20-count pack of AA batteries has the lower price of $0.79 per battery.

Learn more about inequality

here: https://brainly.com/question/11613554

#SPJ2

What is the percentage of 204 over 1015, 1 over 8120, 1 over 5832, and 1 over 6?

Answers

Answer:

204/1015 (irreducible) = 20.1%

1/8120 (irreducible) = 0.01232%

1/5832 (irreducible) = 0.01715%

1/6 (irreducible) = 16.67%

Step-by-step explanation:

The chief business officer of a construction equipment company arranges a loan of $9,300, at 12 1 /8 % interest for 37.5 months. Find the amount of interest. (Round to the nearest cent)

a. $2,761.21


b. $3,583.83


c. $3,523.83


d. $3,722.47

Answers

Answer:

C). $3523.83

Step-by-step explanation:

loan of principles p= $9,300,

at rate R= 12 1 /8 % interest

Rate R = 12.125%

for duration year T = 37.5 months

T= 37.5/12 = 3.125 years

Interest I=PRT/100

Interest I =( 9300*12.125*3.125)/100

Interest I = (352382.8125)/100

Interest I = 3523.83

Interest I= $3523.83

Please help me. What is the y intercept of the graph shown below?

Answers

Answer:

(0,2)

Step-by-step explanation:

the point where Oy intercepts the graph has x=0 and y= f(0)

so this is (0,2)

5) Suppose a slice of a 12-inch pizza has an area of 20 square inches. What is the angle of
this slice?

Answers

Answer:

The angle of the slice is 63.64 degrees

Step-by-step explanation:

Here in this question, we are concerned with calculating the angle of the slice.

What we should know are as follows;

1. A pizza is a perfect circular shape

2. A 12-inch pizza means the diameter of the pizza is 12 inches and consequently its radius will be 12/2 = 6 inches

3. A slice of a pizza represents a sector of the circle( a sector is part of a circle bounded by 2 radii and an arc)

Mathematically, to calculate the angle of the slice, we simply use the formula for the area of a sector.

Area of sector = theta/360 * pi * r^2

where Area of sector = 20 square inches

theta is the center angle we are looking for

r is the radius of the pizza which is 6 inches

Plugging these values into the area of sector equation, we have

20 = theta/360 * 22/7 * 6^2

20 = theta/10 * 22/7

22 theta = 10 * 20 * 7

theta = 1400/22

theta = 63.64 degrees which is approximately 64 degrees to the nearest degree

Simplify 3 x times the fraction 1 over x to the power of negative 4 times x to the power of negative 3.

Answers

Answer:

3x^2

Step-by-step explanation:

3 x times the fraction 1 over x to the power of negative 4 => 3x * 1/x^-4

= 3x *x^4 = 3x^5

times x to the power of negative 3 => x^-3

3x^5 * x^-3 = 3x^2

Answer:

3x^2

Step-by-step explanation:

i got it right on the test on god!

Jayden, who burns 345 calories in 45 min
while hiking is preparing for a 6 hour hike.
He uses a special supplement beverage
pack that provides water, needed
electrolytes, and 310 calories. The goal is to
replace roughly 1/3 of the calories burned
while carrying as light a load as possible.
How many packs should he take?

Answers

I think that she should bring three packs. 6 hours=360 minutes. 360/45=8. 345x8=2760. 2760/3=920 and 310x3=930. :)

This question is solved using proportions.

First, we find how many calories he will burn in the hike.Then, we find how many calories he will need to replace, and the number of packs needed.

Doing this, we get that he should take 3 packs.

How many calories he burns in the hike?

In 45 minutes, he burns 345 calories. How many calories in 6*60 = 360 minutes?

45 minutes - 345 calories

360 minutes - x calories

Applying cross multiplication:

[tex]45x = 345*360[/tex]

[tex]x = \frac{345*360}{45}[/tex]

[tex]x = 2760[/tex]

He burns 2760 calories in the hike.

How many calories he wants to replace?

Roughly 1/3, so he have to find one third of 2760, that is:

[tex]\frac{2760}{3} = 920[/tex]

How many packs?

One pack recovers 310 calories, how many packs for 920 calories?

1 pack - 310 calories

x packs - 920 calories

Applying cross multiplication:

[tex]310x = 920[/tex]

[tex]x = \frac{920}{310}[/tex]

[tex]x = 2.97[/tex]

Rounding up, he should take 3 packs.

A similar question is found at https://brainly.com/question/14426926

A researcher wishes to examine the relationship between years of schooling completed and the number of pregnancies in young women. Her research discovers a linear relationship, and the least squares line is: ˆ y = 3 − 5 x y^=3-5x where x is the number of years of schooling completed and y is the number of pregnancies. The slope of the regression line can be interpreted in the following way:
1.) When amount of schooling increases by one year, the number of pregnancies decreases by 4.
2.) When amount of schooling increases by one year, the number of pregnancies increases by 4.
3.) When amount of schooling increases by one year, the number of pregnancies increases by 5.
4.) When amount of schooling increases by one year, the number of pregnancies decreases by 5.

Answers

Answer:

1. When amount of schooling increases by one year, the number of pregnancies will decrease by 4.

Step-by-step explanation:

Regression analysis is a statistical technique which is used for forecasting. It determines the relationship between two variables. It determines the relationship of two or more dependent and independent variables. It is widely used in stats to find trend in the data. It helps to predict the values of dependent and independent variables. In the given question, there is regression equation given. X and Y are considered as dependent variables. When number of schooling increases by 1 year then number of pregnancies will decrease by 4

The hypotenuse of a right triangle is 14 in. If the base
of the triangle is 2 inches determine the
length of the remaining side.
14 in
Х
2 in
O A &
B. 318
O c. 8v3
OD. 112

Answers

Answer:

13.85

Step-by-step explanation:

U use the pythagorean theorem

So 2^2 + x^2 = 14^2

Simplify the equation: 4+x^2=196

--> x^2=192

--> x=13.85

-Hope this helps :)

9514 1404 393

Answer:

  c.  8√3

Step-by-step explanation:

The Pythagorean theorem applies.

  14² = s² + 2²

  s = √(14² -2²) = √192 = 8√3

The length of the remaining side is 8√3.

Fill in the blanks and explain the pattern.

XA, XB, XC, __,__,__

Answers

Answer:

XD,XE,XF

Step-by-step explanation:

XA,XB,XC,XD,XE,XF

IT IS BECAUSE OF THE ALPHABETICAL ORDER AFTER X

The double number lines show the ratio of cups to gallons. How many cups are in 333 gallons? _____ cups

Answers

Answer:

5328 cups.

Step-by-step explanation:

Given that 333 gallons

We know that

1 gallons = 16 cups

1 cups = 0.0625 gallons

Therefore,from the above conversion we can say that

Now by putting the values in the above conversion

333 gallons = 16 x 333 cups

333 gallons = 5328 cups

So , we can say that 333 gallons is equal to 5328 cups.

Thus the answer will be 5328 cups.

Answer:

48 cups(BTW he meant 33 galons, IVE had this before). lol you need to put the double number line image. first u have to divide 64/4 to get 16, Then it says "How many cups are in 3 gallons". There fore, U multiply 16 to 3 to get ur answer "48".

Which of the following could be the equation of the line passing through (8, 3) parallel to y = -2.

Answers

Answer:

y = 3 passes through (8, 3) and is therefore parallel to y = -2

Step-by-step explanation:

Any line parallel to y = -2 is a horizontal one, and it has the same slope (zero) as does y = -2.

We could invent the horizontal line y = 3 (which comes from the point (8, 3) and surmise that it is parallel to the given line y = -2.

Thus, y = 3 passes through (8, 3) and is therefore parallel to y = -2.

In the future, please share any answer choices that are give you.  Thank you.

Find the first term in the sequence when u(subscript)31=197 and d= 10.

Answers

Answer:

197 = 10(31-1) + a

197 = 300 + a

-103 = a

In a survey of 200 publicly-traded companies, the average price-earnings ratio was 18.5 with a standard deviation of 8.2. When testing the hypothesis (at the 5% level of significance) that the average price-earnings ratio has increased from the past value of 16.8, the null and alternative hypotheses would be:________

Answers

Answer:

Null Hypothesis: H0:μ ≤ 16.8

Alternative Hypothesis: Ha: μ > 16.8

Step-by-step explanation:

We are told that affer testing the hypothesis (at the 5% level of significance), that the average price-earnings ratio increased from the past value of 16.8.

It means that the past value was not more than 16.8.

This follows that the null hypothesis is given as;

H0:μ ≤ 16.8

And since it has been discovered that the ratio increased from the past value of 16.8, the alternative hypothesis is;

Ha: μ > 16.8

Select the correct answer from each drop-down menu.
Nirja has 24 marbles. The number of marbles Nirja has is 6 more than the number of marbles Tim has.
If Tim has x marbles, the equation that represents the situation is
The value of x that makes the equation true is
Reset
Next

Answers

Answer:

24 = x+6

x = 18

Step-by-step explanation:

N = 24

T = x

N = x+6  

24 = x+6

Subtract 6 from each side

24-6 = x+6-6

18 = x

Time has 6 marbles

Nirja has 6 more than Tim,

So you can subtract 6 from 24 to find x:

24-6 = x

Or you can add 6 to x to equal 24:

x + 6 = 24

You don't list the choices but it should be one of these.

Solve:

24 - 6 = x

x = 18

You sell tickets at school for fundraisers. You sold car wash tickets, silly string fight tickets and dance tickets – for a total of 380 tickets sold. The car wash tickets were $5 each, the silly sting fight tickets were $3 each and the dance tickets were $10 each. If you sold twice as many silly string tickets as car wash tickets, and you have $1460 total. Write the matrix in the box below. Write the solution set for this system and include any necessary work.

Answers

Answer:

Matrix :

[tex]\begin{bmatrix}1&1&1&|&380\\ 5&3&10&|&1460\\ -2&1&0&|&0\end{bmatrix}[/tex]

Solution Set : { x = 123, y = 246, z = 11 }

Step-by-step explanation:

Let's say that x represents the number of car wash tickets, y represents the number of silly sting fight tickets, and z represents the number of dance tickets. We know that the total tickets = 380, so therefore,

x + y + z = 380,

And the car wash tickets were $5 each, the silly sting fight tickets were $3 each and the dance tickets were $10 each, the total cost being $1460.

5x + 3y + 10z = 1460

The silly string tickets were sold for twice as much as the car wash tickets.

y = 2x

Therefore, if we allign the co - efficients of the following system of equations, we get it's respective matrix.

System of Equations :

[tex]\begin{bmatrix}x+y+z=380\\ 5x+3y+10z=1460\\ y=2x\end{bmatrix}[/tex]

Matrix :

[tex]\begin{bmatrix}1&1&1&|&380\\ 5&3&10&|&1460\\ -2&1&0&|&0\end{bmatrix}[/tex]

Let's reduce this matrix to row - echelon form, receiving the number of car wash tickets, silly sting fight tickets, and dance tickets,

[tex]\begin{bmatrix}5&3&10&1460\\ 1&1&1&380\\ -2&1&0&0\end{bmatrix}[/tex] - Swap Matrix Rows

[tex]\begin{bmatrix}5&3&10&1460\\ 0&\frac{2}{5}&-1&88\\ -2&1&0&0\end{bmatrix}[/tex] - Cancel leading Co - efficient in second row

[tex]\begin{bmatrix}5&3&10&1460\\ 0&\frac{2}{5}&-1&88\\ 0&\frac{11}{5}&4&584\end{bmatrix}[/tex] - Cancel leading Co - efficient in third row

[tex]\begin{bmatrix}5&3&10&1460\\ 0&\frac{11}{5}&4&584\\ 0&\frac{2}{5}&-1&88\end{bmatrix}[/tex] - Swap second and third rows

[tex]\begin{bmatrix}5&3&10&1460\\ 0&\frac{11}{5}&4&584\\ 0&0&-\frac{19}{11}&-\frac{200}{11}\end{bmatrix}[/tex] - Cancel leading co - efficient in row three

And we can continue, canceling the leading co - efficient in each row until this matrix remains,

[tex]\begin{bmatrix}1&0&0&|&\frac{2340}{19}\\ 0&1&0&|&\frac{4680}{19}\\ 0&0&1&|&\frac{200}{19}\end{bmatrix}[/tex]

x = 2340 / 19 = ( About ) 123 car wash tickets sold, y= 4680 / 19 =( About ) 246 silly string fight tickets sold, z = 200 / 19 = ( About ) 11 tickets sold

��2222 is the diameter of a circle. The coordinates are �(−2, −3) and �(−12, −5). At what coordinate is the center of the circle located? A. (5, 1) B. (−5, −1) C. (−4, −7) D. (−7, −4)

Answers

Answer:

D ). (-7,-4)

Step-by-step explanation:

To locate the position or the location of the centre of the circle we have to bear in mind that the center of the circle is the midpoint of the diameter line.

Formula for midpoint of a line is given below

Midpoint= (X1+x2)/2 ,(y1+y2)/2

Where X1= -2,y1= -3

X2= -12, y2= -5

The midpoint= (-2+(-12))/2,(-3+(-5))/2

Midpoint= (-2-12)/2,(-3-5)/2

Midpoint= (-14)/2,(-8)/2

Midpoint=( -7,-4)

The center of the circle is located at the point (-7,-4)

Find the product of
the sum of
3/5 and 1%
and​

Answers

Answer:

3/500

Step-by-step explanation:

3/5 x 1%

=> 3/5 x 1/100

=> 3/500

Hope it helps you

The coffee cups can hold 7/9 of a pint of liquid. If Emily pours 2/3 of a pint of coffee into a cup,how much milk can a customer add? PLZ HELP!​

Answers

Answer:

1/9

Step-by-step explanation:

easy 2/3 is equivalent to 6/9. So there is 1/9 of a pint left

The Tran family and the Green family each used their sprinklers last summer. The water output rate for the Tran family's sprinkler was 35L per hour. The water output rate for the Green family's sprinkler was 40L per hour. The families used their sprinklers for a combined total of 50 hours, resulting in a total water output of 1900L. How long was each sprinkler used?

Answers

Answer:

Tran family's sprinkler was used for 20 hours

Green's  family's sprinkler was used for 30 hours

Step-by-step explanation:

Let the hours for which Tran family's sprinkler used is x hours

water output rate for the Tran family's sprinkler = 35L per hour

water output from  Tran family's sprinkler in x hours = 35*x L = 35x

Let the hours for which Green family's sprinkler used is y hours

water output rate for the Green family's sprinkler = 40L per hour

water output from  Green family's sprinkler in x hours = 40*y L = 40y

Given

The families used their sprinklers for a combined total of 50 hours

thus

x + y = 50 -------------------equation 1

y = 50-x

total water output of 1900L

35x+40y = 1900  -------------------equation 1

using  y = 50-x in equation 2, we have

35x + 40(50-x) = 1900

35x + 2000 - 40x = 1900

=> -5x = 1900 - 2000 = -100

=> x = -100/-5 = 20

y = 50-20 = 30

Thus,

Tran family's sprinkler was used for 20 hours

Green's  family's sprinkler was used for 30 hours

evaluate the expression 4x^2-6x+7 if x = 5

Answers

Answer:

77

Step-by-step explanation:

4x^2-6x+7

Let x = 5

4* 5^2-6*5+7

4 * 25 -30 +7

100-30+7

7-+7

77

The volume of a cone varies jointly with the base (area) and the height. The volume is 12.5 ft3 when the base (area) is 15 ft2 and the height is 212 ft. Find the volume of the cone (after finding the variation constant) when the base (area) is 12 ft2 and the height is 6 ft

Answers

The volume of the cone, when the base area is 12 ft² and the height is 6 ft, is approximately 24 ft³.

To find the volume of the cone when the base area is 12 ft² and the height is 6 ft, we need to first determine the variation constant relating the volume, base area, and height.

Let's denote the volume of the cone as V, the base area as A, and the height as h. According to the problem, the volume varies jointly with the base area and the height.

Therefore, we can write the following equation:

V = k * A * h

Here k is the variation constant we want to find.

Given one set of values: when A = 15 ft² and h = 2 1/2 ft, V = 12.5 ft³.

Substitute these values into the equation and solve for k:

12.5 ft³ = k * 15 ft² * (2.5 ft)

Now, we can solve for k:

k = 12.5 ft³ / (15 ft² * 2.5 ft)

k = 0.3333 ft

Now that we have the value of the variation constant (k), we can find the volume when A = 12 ft² and h = 6 ft:

V = k * A * h

V = 0.3333 ft * 12 ft² * 6 ft

V = 23.9996 ft³

Therefore, the volume of the cone is 24 ft³.

Learn more about the volume of the cone here:

brainly.com/question/1578538

#SPJ4

The correct question is as follows:

The volume of a cone varies jointly with the base (area) and the height. The volume is 12.5 ft³ when the base (area) is 15 ft² and the height is 2 1/2 ft. Find the volume of the cone (after finding the variation constant) when the base (area) is 12 ft² and the height is 6 ft.

The Masim family’s monthly budget is shown in the circle graph provided in the image. The family has a current monthly income of $5,000. How much money do they spend on food each month? A. $250 B. $500 C. $750 D. 1,100 Please show ALL work! <3

Answers

Answer:

C. $750

Step-by-step explanation:

The amount of money to be spent monthly on food = percentage covered by food in the circle ÷ 100% × total monthly income

= [tex] \frac{15}{100}*5000 [/tex]

[tex] = \frac{15}{1}*50 [/tex]

[tex] 15*50 = 750 [/tex]

Amount of money spent each month by the Masims is $750.

Consider the given function and the given interval.

f(x) = 8 sin x - 4 sin 2x, [0,pi]

(a) Find the average value f ave of f on the given interval.

(b) Find c such that f ave = f(c).

Answers

(a) The average value of f(x) on the closed interval [0, π] is

[tex]\displaystyle\frac1{\pi-0}\int_0^\pi f(x)\,\mathrm dx = \frac1\pi\int_0^\pi(8\sin(x)-4\sin(2x))\,\mathrm dx = \boxed{\frac{16}\pi}[/tex]

(b) By the mean value theorem, there is some c in the open interval (0, π) such that f(c) = 16/π. Solve for c :

8 sin(c) - 4 sin(2c) = 16/π

8 sin(c) - 8 sin(c) cos(c) = 16/π

sin(c) - sin(c) cos(c) = 2/π

Use a calculator to solve this. You should get two solutions, c ≈ 1.2382 and c ≈ 2.8081.

Find the sum of 1 + 3/2 + 9/4 + …, if it exists. This is infinite series notation. The answer is NOT 4.75.

Answers

Answer:

D

Step-by-step explanation:

First, this looks like a geometric series. To determine whether or not it is, find the common ratio. To do this, we can divide the second term and the first term, and then divide the third term and the second term. If they equal to same, then this is indeed a geometric series.

[tex](3/2)/(1)=3/2\\(9/4)/(3/2)=(9/4)(2/3)=18/12=3/2[/tex]

Therefore, this is indeed a geometric series with a common ratio of 3/2.

With just this, we can stop. This is because since the common ratio is greater than one, each subsequent value is going to be bigger than the previous one. Because of this, the series will not converge. Therefore, the series has no sum.

To see this more clearly, imagine a few more terms:

1, 1.5, 2.25, 3.375, 5.0625...

Each subsequent term will just increase. The sum will not converge.

Answer:

No Sum --- it doesn't exist.

Step-by-step explanation:

The partial sums get arbitrarily large--the go to infinity.

The geometric series you are trying to sum has common ratio = 3/2.

The sum of the infinite series exists only when |common ratio| < 1.

The formula for the partial sum of n terms is (r^(n+1) - 1) / (r - 1) = (1.5^(n+1) - 1) / 0.5, or in decimals instead of fraction.. i.e. 1 + 1.5 + 2.25 + 5.0525 + 25.628 + 656.840..... therefore It would take a long time but you'd be adding up forever and goes to infinity.

Other Questions
Vanessa uses the expressions (3x2 + 5x + 10) and (x2 3x 1) to represent the length and width of her patio. Which expression represents the area (lw) of Vanessas patio? Racquel deposited $40,000 in an account paying 4.8% annual interest in 2009. She now wants to make four equal annual withdrawals from the account starting with the first withdrawal in 2021. Racquel also wants to have exactly $60,000 left in the account in 2026. How large can each of the annual withdrawals be? How did humans adapt to the conditions of the last Ice Age? Select the three correct answers. A. They built permanent dwellings from wood and stone. B. They crossed newly exposed land bridges to warmer regions. C. They followed migrating animal herds to ensure their food supply. D. They used fire, portable huts, and animal furs to stay warm. E. They settled in caves and waited for warmer temperatures to return. Help, please.. what is SI unit System ? why has SI system been developed ? Give reasons Astrid is in charge of building a new fleet of ships. Each ship requires 40 4040 tons of wood, and accommodates 300 300300 sailors. She receives a delivery of 4 44 tons of wood each day. The deliveries can continue for 100 100100 days at most, afterwards the weather is too bad to allow them. Overall, she wants to build enough ships to accommodate at least 2100 21002100 sailors. What are the two main categories of mutations that occur in humans? Name and define the two types? The ____ environment can greatly influence the course of human events.A. polytheismB. physicalC. GregorianD. city-stateE. artifact Choose a strategy to clone genes known only by a mutant phenotype in organisms with highly efficient transformation procedures and relatively small genomes. a. cloning by complementation b. direct sequencing of the mutant genome c. PCR of the gene and transformation of an organism d. cloning using a specific cloning vector hat is grammatically incorrect with the following sentence? This question is very difficult to workout. a. The suffix "out" is prepositional, and you cannot end a sentence with a preposition. b. There is an incorrect choice of word: "hard" should be used instead of "difficult". c. The phrasal verb "work out" should be two words, not one. d. There is nothing grammatically incorrect with this sentence. Find the value of |5| - 4(32 - 2). How does antifreeze rely on colligative properties to work?A. It causes the vapor pressure to increase and the increased outward pressure of the gasoline vapors prevents it from solidifying into ice.B. It causes freezing point elevation which raises the temperature of the gasoline without letting it freeze.C. It causes freezing point depression which requires the gasoline to reach a much lower temperature before freezing.D. It causing boiling point elevation, which raises the temperature of the gasoline without letting it boil so that it can resist the cold.WILL GIVE BRAINLIEST FOR CORRECT!!! 10. CORRIGE el mal empleo de los pronombresen estas frases:1- me lo repita, por favor. 2-Delegado les ordena a tus companeros que conLesten la encuesta con seriedad. 3-La lavadora quiero la tener antes del sbado.4-Esta catedral es impresionante. Me alegro de la haber viste.5-El viaje fue tan interesante que todos estaban satisfechos de haber podidolo hacer6-stbamelo contando Rueben is in a biology class and the professor tells the students to find a lab partner. Rueben immediately approaches an Asian classmate because Rueben believes that all Asian students are naturally gifted in mathematics and science. Ruben's mental shortcut is an example of a(n) ________. The term _____ began to be used during the Iraq War and described the forces opposed to American involvement. Members are typically organized into covert groups that engage in organized campaigns of extreme violence. Lourdes considers herself lucky. Immigration has redefined her, and she is grateful. Unlike her husband, she welcomes her adopted language, its possibilities for reinvention. Lourdes relishes winter most of allthe cold scraping sounds on sidewalks and windshields, the ritual of scarves and gloves, hats and zip-in coat linings. Its layers protect her. She wants no part of Cuba, no part of its wretched carnival floats creaking with lies, no part of Cuba at all, which Lourdes claims never possessed her. Which sentence is an example of magic realism? Unlike her husband, she welcomes her adopted language, its possibilities for reinvention. Lourdes relishes winter most of allthe cold scraping sounds on sidewalks and windshields, the ritual of scarves and gloves, hats and zip-in coat linings. Its layers protect her. She wants no part of Cuba, no part of its wretched carnival floats creaking with lies, no part of Cuba at all, which Lourdes claims never possessed her. Place the following events in order to trace the development of human societies.Humans survived by hunting andAdvanced civilizationsPeople could farm and store aThe Neolithic Revolutiongatheringdevelopedoccurredsurplus of food.IntroDone Please help with 2 and 3 for maths Hawkins Poultry Farms is considering the purchase of feeding equipment that costs $139,000 and will produce annual cash flows of approximately $36,000 for five years. The equipment is expected to be sold at the end of five years for $40,000. What is the net present value of the proposed investment What is the correct conjugation for volver if I want to use the condicional mood Ustedes vuelvan a visitarnos? What type of chemist exclusively studies most carbon compounds?-biochemist-physical chemist-organic chemist-inorganic chemist